6
$\begingroup$

I'm searching for the solution $\bar{a}$ to the system of equations $\bar{e}_1 = B\bar{a}$ given by

\begin{equation} \left[\begin{array} & 1 \\ 0 \\ \vdots \\ 0 \\ 0 \end{array}\right] = \left[\begin{array} & b_1 & 0 & \cdots & 0 & 0 \\ b_2 & b_1 & \cdots & 0 & 0 \\ \vdots & \vdots & \ddots & \vdots & \vdots \\ b_{n-1} & b_{n-2} & \cdots & b_1 & 0 \\ b_n & b_{n-1} & \cdots & b_2 & b_1 \end{array}\right] \left[\begin{array} & a_1 \\ a_2 \\ \vdots \\ a_{n-1} \\ a_n \end{array}\right] \end{equation}

I can easily solve this system of equations algorithmically, but I am curious whether there is a straightforward way to solve it analytically.

The coefficient matrix $B$ has a clear structure - it's lower triangular and all elements along the diagonal, subdiagonal, and so on are identical. I am sure that there is a name for such a structure and most likely it has been studied before, but unfortunately I haven't been able to find where to look to find more information about it in order to invert it or otherwise solve this system of equations.

I've solved for the first few $a_j$ but there hasn't been a clear pattern. If there is a name for matrices like $B$ that I can reference, or if anyone can point me to a closed-form solution for each $a_j$ in terms of $b_k$ for $1 \le k \le j$, I'd appreciate it.

$\endgroup$
1
  • 4
    $\begingroup$ $B$ is a lower triangular Toeplitz matrix. $\endgroup$ Commented 15 hours ago

2 Answers 2

4
$\begingroup$

I hand calculated the first 6 with forward substitution and found that if you let $\mathcal{M}_{i,j}$ be the set of ordered pairs of integers $(m_1,m_2,...,m_{j-1})$ such that $\sum\limits_{k=1}^{j-1}m_k=i+j-2$ and each $m_k\ge 2$, then for $i>1$ you will have $a_i=\sum\limits_{j=2}^i\frac{(-1)^{j+1}}{b_1^j}\big[\sum\limits_{(m_1,...,m_{j-1})\in \mathcal{M}_{i,j}}b_{m_1}b_{m_2}...b_{m_{j-1}}\big]$. Of course for $i=1$ you have $a_1=\frac{1}{b_1}$.

Here is a proof by strong induction I just wrote.

The base cases for $i=1,2$ are easy to show with forward substitution. Now let $i\ge2$ and assume the hypothesis holds for all $k\le i$, then $$a_{i+1}=-\frac{1}{b_1}\sum\limits_{k=1}^{i}b_{i+2-k}a_k$$$$=-\frac{1}{b_1}\Big(\frac{b_{i+1}}{b_1}+\sum\limits_{k=2}^{i}b_{i+2-k}\sum\limits_{j=2}^k\frac{(-1)^{j+1}}{b_1^j}\big[\sum\limits_{(m_1,...m_{j-1})\in\mathcal{M}_{k,j}}b_{m_1}b_{m_2}...b_{m_{j-1}}\big]\Big)$$$$=-\frac{b_{i+1}}{b_1^2}+\sum\limits_{k=2}^i\sum\limits_{j=2}^k\frac{(-1)^{j+2}}{b_1^{j+1}}\big[\sum\limits_{(m_1,...m_{j-1})\in\mathcal{M}_{k,j}}b_{m_1}b_{m_2}...b_{m_{j-1}}b_{i+2-k}\big]$$

Notice for each $2\le k\le i$ and $2\le j\le k$, it is clear that $(m_1,...,m_{j-1})\in\mathcal{M}_{k,j}$ implies $(m_1,...,m_{j-1},i+2-k)\in\mathcal{M}_{i+1,j+1}$. Furthermore, for each $2\le j\le i$ and $(m_1,...,m_j)\in\mathcal{M}_{i+1,j+1}$, since each entry of the ordered pair is greater than or equal to $2$ and the entries sum to $i+j$, we must have $m_j\le i+j-2(j-1)=i-j+2$, implying $m_j=i+2-k$ for some unique $k$ with $j\le k \le i$, and of course $(m_1,...,m_{j-1})\in\mathcal{M}_{k,j}$. It follows that the previous double sum is equivalent to $$\sum\limits_{j=2}^i\frac{(-1)^{j+2}}{b_1^{j+1}}\big[\sum\limits_{(m_1,...m_{j})\in\mathcal{M}_{i+1,j+1}}b_{m_1}b_{m_2}...b_{m_j}\big]=\sum\limits_{j=3}^{i+1}\frac{(-1)^{j+1}}{b_1^{j}}\big[\sum\limits_{(m_1,...m_{j-1})\in\mathcal{M}_{i+1,j}}b_{m_1}b_{m_2}...b_{m_{j-1}}\big]$$ Substituting back into the formula for $a_{i+1}$ and noting that $\mathcal{M}_{i+1,2}=\{(i+1)\}$, we have $$a_{i+1}=-\frac{b_{i+1}}{b_1^2}+\sum\limits_{j=3}^{i+1}\frac{(-1)^{j+1}}{b_1^{j}}\big[\sum\limits_{(m_1,...m_{j-1})\in\mathcal{M}_{i+1,j}}b_{m_1}b_{m_2}...b_{m_{j-1}}\big]$$$$=\sum\limits_{j=2}^{i+1}\frac{(-1)^{j+1}}{b_1^{j}}\big[\sum\limits_{(m_1,...m_{j-1})\in\mathcal{M}_{i+1,j}}b_{m_1}b_{m_2}...b_{m_{j-1}}\big]$$ This proves the result.

$\endgroup$
4
$\begingroup$

Rephrasing slightly,

$$ \begin{bmatrix} b_0 & 0 & \cdots & 0 & 0 \\ b_1 & b_0 & \cdots & 0 & 0 \\ \vdots & \vdots & \ddots & \vdots & \vdots \\ b_{n-2} & b_{n-3} & \cdots & b_0 & 0 \\ b_{n-1} & b_{n-2} & \cdots & b_1 & b_0 \end{bmatrix} \begin{bmatrix} a_0 \\ a_1 \\ \vdots \\ a_{n-2} \\ a_{n-1} \end{bmatrix} = \begin{bmatrix} 1 \\ 0 \\ \vdots \\ 0 \\ 0 \end{bmatrix} $$

where $b_0 \neq 0$. Note that the LHS is the (discrete) convolution of sequences $a$ and $b$. One would like to find the sequence $a$ of length $n$ such that $a \star b = \delta$, where $\delta$ is a unit pulse. In signal processing, one would call this deconvolution. In communications engineering, one would call this channel equalization. Taking the $\cal{Z}$-transforms of both sides of $a \star b = \delta$,

$$ \left( \sum_{k=0}^{n-1} a_k z^{-k} \right) \left( \sum_{\ell=0}^{n-1} b_{\ell} z^{-\ell} \right) = 1 $$

and, thus, $a_k$ is the coefficient of $z^{-k}$ in $\left( \sum\limits_{\ell=0}^{n-1} b_{\ell} z^{-\ell}\right)^{-1}$.


Related


$\endgroup$

You must log in to answer this question.

Start asking to get answers

Find the answer to your question by asking.

Ask question

Explore related questions

See similar questions with these tags.